(5 intermediate revisions by the same user not shown)
Line 50: Line 50:
  
 
d)  
 
d)  
[[ Image:Pro1_solution11_2015_Aug.jpg]]<br />
+
[[ Image:Pro1_solution1_2015_Aug.jpg]]<br />
  
 
e) We are likely to see quantization artifact in dark region.
 
e) We are likely to see quantization artifact in dark region.
Line 61: Line 61:
 
\gamma=log_{\frac{R}{255}}{(R^{\alpha})}=\frac{ln{(R^{\alpha})}}{ln{\frac{R}{255}}}=\frac{\alpha{ln{R}}}{ln{R}-ln{255}}
 
\gamma=log_{\frac{R}{255}}{(R^{\alpha})}=\frac{ln{(R^{\alpha})}}{ln{\frac{R}{255}}}=\frac{\alpha{ln{R}}}{ln{R}-ln{255}}
 
</math>
 
</math>
 +
 +
<span style="color:green"> <math>\gamma </math> should be 1. </span>
  
 
b)  
 
b)  
Line 156: Line 158:
 
[[ Image:Pro1_2015_Aug.PNG ]]<br />
 
[[ Image:Pro1_2015_Aug.PNG ]]<br />
  
e) Gamma correction an quantization will create an effect of dynamic range compression for pixels with small values. This will create dark block of shadings in a gradient region instead of a smooth transition.
+
e) Gamma correction a quantization will create an effect of dynamic range compression for pixels with small values. This will create dark block of shadings in a gradient region instead of a smooth transition.
  
 
----
 
----
===Related Problem===
+
[[ECE-QE_CS5-2015|Back to QE CS question 1, August 2013]]
Consider a color imaging device that takes input values of <math> (r,g,b) </math> and produces ouput <math> (X,Y,Z)</math> values given by
+
  
<math>
 
\left[ {\begin{array}{*{20}{c}}
 
X\\
 
Y\\
 
Z
 
\end{array}} \right] = \left[ {\begin{array}{*{20}{c}}
 
a&b&c\\
 
d&e&f\\
 
g&h&i
 
\end{array}} \right]\left[ {\begin{array}{*{20}{c}}
 
r^\alpha\\
 
g^\alpha\\
 
b^\alpha
 
\end{array}} \right]
 
</math>
 
 
a) Calculate the white point of the device in chromaticity coordinates.
 
 
b) What are the primaries associated with the r,g, and b components respectively?
 
 
c) What is the gamma of the device?
 
 
d) Draw the region on the chromaticity diagram corresponding to <math> r < 0, g > 0, b > 0</math>.
 
----
 
[[ECE-QE_CS5-2015|Back to QE CS question 1, August 2013]]
 
 
[[ECE_PhD_Qualifying_Exams|Back to ECE QE page]]:
 
[[ECE_PhD_Qualifying_Exams|Back to ECE QE page]]:

Latest revision as of 23:34, 3 December 2015


ECE Ph.D. Qualifying Exam in Communication Networks Signal and Image processing (CS)

August 2015, Part 1

Part 1 , 2

Solution 1

a) $ \gamma=1 $

b) $ \left( \begin{array}{c} X_r\\ Y_r \\ Z_r \end{array} \right)= \left( \begin{array}{ccc} a & b & c \\ d & e & f \\ g & h & i \end{array} \right) \left( \begin{array}{c} 1 \\ 0 \\ 0 \end{array} \right)= \left( \begin{array}{c} a \\ d \\ g \end{array} \right) $
So $ (x_r,y_r)=(\frac{X_r}{X_r+Y_r+Z_r}, \frac{Y_r}{X_r+Y_r+Z_r})=(\frac{a}{a+d+g},\frac{d}{a+d+g}) $
Similarly $ (x_g,y_g)=(\frac{b}{b+e+h},\frac{e}{b+e+h}) $, $ (x_b,y_b)=(\frac{c}{c+f+i},\frac{f}{c+f+i}) $

c) The white point of the device is when the input $ [R, G, B] = [1, 1, 1] $

$ (x_w,y_w)=(\frac{a+b+c}{a+b+c+d+e+f+g+h+i},\frac{d+e+f}{a+b+c+d+e+f+g+h+i}) $

d) Pro1 solution1 2015 Aug.jpg

e) We are likely to see quantization artifact in dark region.

Solution 2:

a) $ \frac{R}{255}^\alpha=r_{linear}\\ \Rightarrow \gamma=log_{\frac{R}{255}}{(R^{\alpha})}=\frac{ln{(R^{\alpha})}}{ln{\frac{R}{255}}}=\frac{\alpha{ln{R}}}{ln{R}-ln{255}} $

$ \gamma $ should be 1.

b)

$ P_r= \left( \begin{array}{ccc} a & b & c \\ d & e & f \\ g & h & i \end{array} \right) \left( \begin{array}{ccc} 1 \\ 0 \\ 0 \end{array} \right) = \left( \begin{array}{ccc} a \\ d \\ g \end{array} \right) \\ \Rightarrow x_r=\frac{a}{a+d+g} , y_r=\frac{d}{a+d+g} \\ P_g= \left( \begin{array}{ccc} a & b & c \\ d & e & f \\ g & h & i \end{array} \right) \left( \begin{array}{ccc} 0 \\ 1 \\ 0 \end{array} \right) = \left( \begin{array}{ccc} b \\ e \\ h \end{array} \right) \\ \Rightarrow x_g=\frac{b}{b+e+h} , y_g=\frac{e}{b+e+h} \\ P_b= \left( \begin{array}{ccc} a & b & c \\ d & e & f \\ g & h & i \end{array} \right) \left( \begin{array}{ccc} 0 \\ 0 \\ 1\end{array} \right) = \left( \begin{array}{ccc} c \\ f \\ i \end{array} \right) \\ \Rightarrow x_g=\frac{c}{c+f+i} , y_g=\frac{f}{c+f+i} $

c)

$ W= \left( \begin{array}{ccc} a & b & c \\ d & e & f \\ g & h & i \end{array} \right) \left( \begin{array}{ccc} 1 \\ 1 \\ 1\end{array} \right) = \left( \begin{array}{ccc} a+b+c \\ d+e+f \\ g+h+i \end{array} \right) \\ \Rightarrow x_g=\frac{a+b+c}{a+b+c+d+e+f+g+h+i} , y_g=\frac{d+e+f}{a+b+c+d+e+f+g+h+i} $

d) Pro1 2015 Aug.PNG

e) Gamma correction a quantization will create an effect of dynamic range compression for pixels with small values. This will create dark block of shadings in a gradient region instead of a smooth transition.


Back to QE CS question 1, August 2013

Back to ECE QE page:

Alumni Liaison

Ph.D. on Applied Mathematics in Aug 2007. Involved on applications of image super-resolution to electron microscopy

Francisco Blanco-Silva